Last visit was: 26 Apr 2024, 12:35 It is currently 26 Apr 2024, 12:35

Close
GMAT Club Daily Prep
Thank you for using the timer - this advanced tool can estimate your performance and suggest more practice questions. We have subscribed you to Daily Prep Questions via email.

Customized
for You

we will pick new questions that match your level based on your Timer History

Track
Your Progress

every week, we’ll send you an estimated GMAT score based on your performance

Practice
Pays

we will pick new questions that match your level based on your Timer History
Not interested in getting valuable practice questions and articles delivered to your email? No problem, unsubscribe here.
Close
Request Expert Reply
Confirm Cancel
SORT BY:
Date
Tags:
Show Tags
Hide Tags
avatar
Intern
Intern
Joined: 04 Oct 2013
Posts: 6
Own Kudos [?]: 18 [7]
Given Kudos: 5
Send PM
avatar
Intern
Intern
Joined: 04 Oct 2013
Posts: 6
Own Kudos [?]: 18 [3]
Given Kudos: 5
Send PM
avatar
Intern
Intern
Joined: 04 Oct 2013
Posts: 6
Own Kudos [?]: 18 [3]
Given Kudos: 5
Send PM
avatar
Intern
Intern
Joined: 04 Oct 2013
Posts: 6
Own Kudos [?]: 18 [2]
Given Kudos: 5
Send PM
Chinese Remainder Theorem [#permalink]
2
Kudos
Let’s understand this theorem with an example:

Q.5) – Rahul has certain number of cricket balls with him. If he divides them into 4 equal groups, 2 are left over. If he divides them into 7 equal groups, 6 are left over. If he divides them into 9 equal groups, 7 are left over. What is the smallest number of cricket balls could Rahul have?

Let N be the number of cricket balls.

N = 2(mod4) ————–> equation 1

N = 6(mod7) ————–> equation 2 &

N = 7(mod9) ————–> equation 3.

From N=2(mod4) we get, N=4a+2

Substituting this in equation 2, we get the following equation:

4a + 2 = 6(mod7)

Therefore, 4a = 4(mod7)

Hence, 2 x 4a = 2 x 4(mod7)

This gives us a = 1(mod7)

Hence a = 7b+1.

Plugging this back to N=4a+2, we get….

N = 28b + 6

Substituting this to equation 2;

28b + 6 = 7(mod9)

28b = 1(mod9)

Therefore, b=1(mod9)

Hence b = 9c + 1.

Substituting this back to equation N=28b+6;

N = 28(9c+1) + 6

N = 252c + 34

The smallest positive value of N is obtained by setting c=0.

It gives us N = 34

TB – All prime numbers greater than 3 can be expressed as 6K+1 or 6K-1, this is another important result. You would be using this result a lot when it comes to number system problems...
avatar
Intern
Intern
Joined: 04 Oct 2013
Posts: 6
Own Kudos [?]: 18 [2]
Given Kudos: 5
Send PM
Concept of Cyclicity [#permalink]
2
Kudos
Q) If n is a positive integer, what is the remainder when [7(8n+3) + 2] is divided by 5?

This problem can be easily solved with concept of cyclicity.

You should realize that unit digit of all numbers raised to powers start repeating itself.

For example, the cyclicity of 7 is 4. This means that the unit digits repeat after on every 4th power. See for yourself below:

71 = 7 75 =7 79=7

42 = 9 76= 9 710=9

73 = 3 77 =3 711=3

74 = 1 78 =1

In this question, consider n=1.

You are looking at unit digit of 7^11 which will be 3 from concept of cyclicity.

Add two to it.

Therefore, last digit will be 5 for [7(8n+3) + 2]. Hence it’s completely divisible by 5.
avatar
Intern
Intern
Joined: 04 Oct 2013
Posts: 6
Own Kudos [?]: 18 [1]
Given Kudos: 5
Send PM
Concept of negative remainder [#permalink]
1
Kudos
Remainder can never be negative; its minimum value can only be 0.

Consider an example of -30 / 7. Here, remainder is 5.

It would not be (-28 – 2 / 7), but [(-35+5)/7]

When you divide, you will get remainder of -2. Since remainder can never be negative, we subtract it from quotient, here 7 – 2 = 5.

Negative remainder is useful when you are trying to solve a problem with higher power.

Consider an example…….

Q) Find the remainder for 7^7^7 is divided 32

32 can be factored into 8 and 4,

Therefore, we will divide the question into two parts:

Remainder [7^7^7/8] x Remainder [7^7^7/4]


which gives: Remainder [(-1) ^7^7/8] x Remainder [3^7^7/4] [(-1) raised to odd power is -1 and raised to even power is 1]

Remainder [(-1)/8] x Remainder [(-1)/4]…. which gives

———-> 7 x 3 = 21
User avatar
Non-Human User
Joined: 09 Sep 2013
Posts: 32688
Own Kudos [?]: 822 [0]
Given Kudos: 0
Send PM
Re: Euler's Remainder Theorem [#permalink]
Hello from the GMAT Club BumpBot!

Thanks to another GMAT Club member, I have just discovered this valuable topic, yet it had no discussion for over a year. I am now bumping it up - doing my job. I think you may find it valuable (esp those replies with Kudos).

Want to see all other topics I dig out? Follow me (click follow button on profile). You will receive a summary of all topics I bump in your profile area as well as via email.
GMAT Club Bot
Re: Euler's Remainder Theorem [#permalink]
Moderator:
Math Expert
92948 posts

Powered by phpBB © phpBB Group | Emoji artwork provided by EmojiOne